SCI. HOBBYIST
ELECTRICITY MISCON.
GOOD STUFF
NEW
SEARCH
 

"ELECTRICITY" MISCONCEPTIONS IN K-6 TEXTBOOKS - - -William J. Beaty

LOADING BIG TEXTFILE...

THIS IS PAGE 2 OF 'ELECTRICITY MISCONCEPTIONS'
SCROLL DOWN


   "It is dangerous to be right in matters on which the 
   established men are wrong." - Voltaire (1752)










































BEN FRANKLIN SHOULD HAVE SAID ELECTRONS ARE POSITIVE? Wrong.


Many authors bemoan the fact that Ben Franklin labeled "resinous electricity" as negative, and "vitreous electricity" as positive. By choosing the polarities this way, Franklin forces us to say that electrons carry a charge of negative electricity. Because of Franklin's decision, we must name the electric currents in solid metals as flows of NEGATIVE charge rather than positive charge.

Did Franklin make a mistake? Should he have defined the electron to be positive? ABSOLUTELY NOT. In fact it's a blessing, since these flows of negative charge aren't inherently confusing. Franklin's choice of polarity fortunately helps reveal the true source of confusion: common and widespread misconceptions about electrons and "electricity."

If Franklin had instead chosen the electrons to be positive, then we might never confront the real problem. If electrons weren't negative, we'd easily ignore our misconceptions, and we'd end up with only an illusion of understanding. Yet also we'd still have all sorts of niggling unanswered questions caused by the misconceptions. Fortunately the backwards negative electrons rub our noses in the problem, forcing our questions grow into something far more than just "niggling!"

The solution isn't to ignore our discomfort and pretend that we understand electricity. The solution is to confront the source of our discomfort. If we dislike negative currents, or if we find them to be confusing or wrong, it's because our misconceptions are inside us, fighting back. Loose our misconceptions, and we lose our confusion.

What are the misconceptions? Here is a list:

  1. All electric currents are flows of electrons. Wrong.
  2. "Electricity" is made of electrons, not protons. Nope.
  3. Electrons are a kind of energy particle. Wrong.
  4. "Electricity" carries zero mass because electrons have little mass. No.
  5. Positive charge is really just a loss of electrons. Wrong.
  6. Positive charge cannot flow. Totally wrong.
  7. To create "static" charge, we move the electrons. Not always.
These seven statements are misconceptions. We have Ben Franklin to thank for rubbing our noses in this fact. If he'd chosen the polarities so electrons came out positive, we'd be much more comfortable. We might never even notice our errors.

Here are the corrections for the above seven mistakes:
1. All electric currents are flows of electrons? Wrong.
Electric currents are not always flows of electrons, they can be flows of any type of electric charge. The type of charge-carrier depends on the type of conductor. Both protons and electrons possess exactly the same amount of 'electricity.' If either the protons or the electrons flow, that flow is an electric current. Yes, in solid metals, electrons do the flowing. But this isn't true for other types of conductor. For example, in salt water, in fluorescent bulbs, in the dirt and in human bodies, atoms with extra protons can flow along, and this flow is a genuine electric current. And in battery acid, fuel cell membranes, and in solid ice, the electric current is actually a flow of positive hydrogen ions, also called single protons. (Go and look up "proton conductor," also the Grotthuss process of proton-jumping electric current.)

2. "Electricity" is made of electrons, not protons? Nope.

Charges of "electricity" are carried both by electrons and by protons. These two types of particles have very different weights (mass), but both have exactly the same amount of electric charge. Electrons are easily removed from atoms, while protons usually are stuck to other protons, but that doesn't affect the amount of charge the protons carry. If we remove an electron from a neutral atom, that atom is left with too many protons, and that's the only reason why the atom has an excess of positive electric charge. (Visualize a hydrogen atom, then remove the electron. It's a pure proton!) Positive charge isn't "just a lack of electrons," see #5 below. Instead, *all* positive charges in objects and in circuits are created by protons. Removing an electron just reveals the positive charge which was already hidden in the neutral atom. (Think like this: neutral atoms are made from "cancelled electricity," from equal positives and negatives. If the protons don't equal the electrons, then we'll have some "uncancelled electricity," caused either by having too many positive protons, or by too many negative electrons.)

3. Electrons are a kind of energy particle? Wrong.

Electrons and protons are matter, not energy. A flow of electrons is NOT a flow of energy, it is a flow of matter and a flow of electric charge. Same goes for protons: in acids, the moving protons are electric current, but they're also a genuine matter-flow. And most important: if you have a certain amount of charge in one place, you'll have no clue about the amount of energy present. Charge is not energy. And if charge is flowing along, you won't know anything about the direction or the rate of the energy's flow. Coulombs are not Joules, Amperes are not Watts, and knowing the amount charge does not tell you the amount of energy present. A moving electron does not carry electrical energy along with it as it goes, any more than a moving air molecule carries a sound wave with it. (Charge is the "medium," and electrical energy is wave-energy. Charge can wiggle back and forth, while electrical energy flows only forwards. Charge can move slow, while the energy zooms along at the speed of light.)

4. "Electricity" carries zero mass because electrons have little mass? No.

Quantities of "Electricity" (meaning charge) have weight because charge is part of matter particles. A flow of charge always requires a flow of carrier particles, so electric current must always carry mass with it. Electric currents in a wire are not a flow of energy, they're a flow of electrically charged matter. Ion currents in an electroplating bath are a flow of considerable amounts of matter: electric currents can transport material and deposit matter onto electrodes. However, in normal circuits we rarely notice the moving mass. There are two reasons for this: the flow is circular, so an electric current doesn't need to build up extra mass anywhere. Secondly, the flow is very very slow, so even if the current were moving an enormous amount of mass, we'd never notice this.

5. Positive charge is really just a loss of electrons? Wrong.

Positive charge is not made of "missing electrons." Positive charge is a genuine type of charge in its own right. Take a neutral hydrogen atom, remove one electron, and we now have positive charge: a single lone proton. Not "missing electrons." Yes, whenever protons and electrons are near each other, their charges cancel. Removing the electrons from neutral matter exposes the charge on the protons, and that's probably where this particular misconception originates. Neutral materials contain "canceled-out" protons and electrons in equal quantities. Since neutral atoms receive a positive imbalance of charge when their electrons are removed, is seems like positive charge is nothing but missing electrons. But this is wrong. The protons were already there, just hidden. Removing the electrons just un-canceled them. If we have a handful of protons, we have a handful of positive charge. A proton is not a missing electron, any more than an electron is a "missing proton." And most important: if we have a vacuum (which contains only "missing electrons,") that doesn't imply that positive charges are present. Vacuums are not positively charged, even though a vacuum is, heh, jam-packed full of missing electrons! In circuits (and in everyday matter,) in order to have positive charges, we have to have protons.

6. Positive charge cannot flow? Totally wrong.

Electric currents in a solid metal wires are flows of electrons, but in many other materials both the positive and negative charges can flow. It all depends on the type of conductor. For example, when we receive an electric shock, no electrons flow through our bodies. The electric current inside our tissues is made of positively charged atoms flowing one way and negatively charged atoms flowing the other. The same is true of electrical currents in salt water, in the ground, and in battery electrolyte. When your car battery is supplying 300 amps to the starter motor, 300A worth of +H ions (pure protons) is flowing through the battery acid, plus a tiny amount of negative sulfates flowing the other way. Also, plasmas can have positive ion currents as well as negative electron flows: examples are neon signs, fluorescent lights, camera flashes, and sparks of all kinds. (In hydrogen plasma, protons flow one way and electrons the other.) In liquid metals the major part of electric current is electron-flow, but movable positive metal ions are also flowing in the other direction, causing fluid streams during high amperes. There are even some types of conductors where the entire current is a flow of positive hydrogen ions, +H ions, otherwise known as protons. One common "proton conductor" is ice. Other proton-conductors are used as the solid- acid electrolytes in exotic batteries and, more recently, are employed as proton-conductive solid electrolyte membranes in fuel cells.

7. To create "static" charge, we transfer the electrons? Not always.

"Static" or imbalanced charges can be created by removing electrons from a neutral atom. They can also be created by adding or removing charged atoms from an object, and the removed ions can be negative or positive ions. It's even possible to add or remove bare protons from some materials (after all, protons are the same as H+ positively charged hydrogen atoms present in any acidic surface layer.) If we have some positively-charged water, or ice, or acid, then we probably have too many bare protons (too many H+ ions, not enough -OH ions to neutralize every proton.)
 

I thank Ben Franklin for the discomfort and the controversy he caused by giving the "wrong" polarity to electrons. Without his "mistake," students and teachers would be much more comfortable in their misconceptions, and they might never search for answers.

I try to take my own advice: I always imagine that electric currents in circuits are not flows of electrons, instead they are flows of "charges" or "charged particles." Unless we know the type of conductor involved, we cannot know whether electric currents are composed of electrons moving in a single direction... or whether they're electrons AND positive atoms moving oppositely, or whether it's moving positives and unmoving negative charges. For example, if you receive an electric shock, no electrons flowed through your body. Only charged atoms flowed.

 
 


THE "ELECTRICITY" WHICH FLOWS IN WIRES IS SUPPLIED BY BATTERIES OR GENERATORS? Wrong.

Electric currents in copper wires are a flow of electrons, but these electrons are not supplied by batteries. Generators do not 'generate' any electrons. Instead the electrons come from the wire. In copper wire, copper atoms supply the flowing electrons. The electrons in a circuit were already there before the battery was connected. They were even there before the copper was mined and made into wires! Batteries and generators do not create these electrons, they merely pump them, and the electrons act like a pre-existing fluid which is always found within all wires. In order to understand electric circuits, we must imagine that all the wires are pre-filled with a sort of "liquid electricity." Wires are not like hollow pipes. They're like pre-filled pipes ...with no bubbles allowed.

To clarify this, get rid of the battery. Batteries are too complicated. Instead, use a hand-cranked generator as your power supply. Ask yourself exactly where the flowing "electricity" comes from whenever a generator powers a light bulb. A hand-cranked generator contains a coil and some magnets. When cranked, it takes electrons in from one terminal and simultaneously spits them out the other terminal. At the same time, the generator pushes electrons through the rotating coil of wire inside itself. It also pushes them through the rest of the circuit. So where did these electrons come from? Unlike the situation with a battery-powered circuit, all we have here is wires. Inside the generator is just more wires. Where is the source of this flowing "electricity?"

When we include the generator in the circuit, we find that the circuit is a continuous closed loop, and we can find no single place where the quantities of "electricity" originate. A generator is like a closed-loop pump, but it does not supply the substance being pumped. That "substance" was inside the wires all along. Batteries act like this as well. The liquid between the battery plates is an electrolyte, and electrolytes are conductors full of movable charges. Some batteries contain acid, others are alkaline batteries, and still others use conductive salt water. Flowing charges go through the battery, and no charges build up inside. (Or better, we should say that the electrolyte's own charges can be forced to flow.)

But weren't we all taught during grade-school that "batteries and generators create Current Electricity"? This phrase forms a serious conceptual stumbling block (at least it did for me!) To fix it, get rid of the bogus idea called "Current Electricity". Instead change the statement to read like this:

"Batteries and generators cause electric charge to flow."
To complete the picture, add this: all conductors are always full of movable charge. That's what a conductor is, it's a material which contains movable charge.

A battery or generator is like your heart: it moves blood, but it does not create blood. When a generator stops, or when the metal circuit is opened, all the electrons stop where they are, and the wires remain filled with electric charges. But this isn't unexpected, because the wires were full of vast quantities of charge in the first place.



"ELECTRICITY" IS A PHENOMENON COMPOSED OF ENERGY? Wrong.

Actually, "Electricity" does not exist. The term "electricity" is a catch-all word with many meanings. Unfortunately these meanings are contradictory, and the experts disagree over which one is the standard definition used in physics. This leads to the unsettling fact that there is no single substance or energy called "electricity." When we say "quantity of electricity," we could be talking about quantities of charged particles. But we could also be talking about quantity of energy, quantity of current, or potential, forces, fields, net charge, power, or even about electrical phenomena. All of these are found as separate dictionary definitions of the word "electricity." But current is not power, particles are not fields, and charge is not energy. "Quantity of Electricity" is a meaningless concept because of the contradictory definitions of the word "electricity."

Much of this problem would vanish if we used the word "electricity" only to refer to a field of science or class of phenomena. This is the way we use the words "physics" or "optics," and we never try to measure a quantity of "optics." Then, if we needed to get down to details, we'd never discuss "electricity." Instead we'd use words like "charges," "energy", "current," etc. We do use the word "electricity" this way occasionally. But then we immediately turn around and do the equivalent of teaching our children that optics is a substance, or that physics is a kind of energy. "Optics" is a substance which comes out of the light bulb and passes through the lens, right? And when you ride a bicycle, "physics" comes out of your muscles and makes the wheels turn? That's basically what we're saying whenever we tell kids that "electricity flows in wires," or that "light bulbs convert electricity into light."

Below are a few examples of errors caused by the contradictory meanings.

  • In AC electric circuits the charges wiggle back and forth, but the energy moves continuously forward. This is analogous to the way that sound waves move continuously forward through the air, while the air itself wiggles back and forth. But if we teach our kids that "electricity" is made of electrons, and "electricity" is also energy, then we make a serious error. We unwittingly teach them that the electricity in wires sits in one spot and wiggles, but at the same time the electricity moves forward at nearly the speed of light. Garbage! It's like saying that sound and air are the same thing. And the error is directly traceable to the bogus "electricity" concept.
  • Another: when a battery lights up an incandescent bulb, we explain that the path of electricity is into, then through, then back out of the bulb, and that no electricity is used up. Then we contradict ourselves and say that electricity flows one-way from the battery to bulb, and is totally converted to light. Which meaning is correct? Does the bulb consume the electricity to make light? Or, does every bit of the electricity flow through the lightbulb filament and back out again through the second wire? As far as students are concerned, we've just told them that it does both things at the same time!
  • Another: There are only two forms of electricity, positive electricity and negative. Correct? NO, the two forms of electricity are static and current. NO, there are many forms of electricity: triboelectricity, bioelectricity, myoelectricity, piezoelectricity. NO, electricity is a single form of energy called Electromagnetism. NO, electricity is power, it is watts, not energy.

    Which is right? All and none, because the word "electricity" has multiple contradictory definitions. None of the above statements are right because there is no "electricity" which is charge, energy, power, and class of phenomena all at once. And all the meanings are also correct, because the word "electricity" is commonly used to name all these different things, and these definitions appear in the dictionary. Who are we to argue with The Dictionary? Yet we should distrust the dictionary, since it just innocently records words which people use. Dictionaries aren't scientific references. If people always use the word "electricity" in misleading and contradictory ways, then dictionaries will contain contradictory definitions.


ELECTRICITY IS A TYPE OF EVENT? NO.

Scientists originally had a very clear meaning for the word "electricity." It meant "charge." They would say that electrons carry negative electricity, and protons carry positive electricity. They would say that an electric current is a flow of electricity. Unfortunately the meaning of the word became corrupted in the early 19th century when electric companies started selling electrical energy. They called this energy by the name "electricity." But this is a serious problem. When you turn on the lights in your home, the charges inside the wires wiggle back and forth and don't flow forwards, while the energy races along the wires at almost the speed of light. So ...does the "electricity" sit in one spot and vibrate? Or does "electricity" flow forwards rapidly? Do electric companies sell electricity, or do they merely pump it back and forth? Scientists of old would insist that the electricity stays with the electrons, and therefore the electricity vibrates within the wires without moving forward. They'd say that utility companies don't sell any electricity. Most modern K-12 textbooks disagree.

Clearly charge is not energy, so the word "electricity" is being used above as a single name for two very different things: for electrons and for electromagnetic energy. Bad move.

Somewhere along the line the grade-school textbooks made the problem worse by creating a third meaning. They started teaching that "electricity" was neither the charges nor the energy. Instead, "electricity" was the motion of the charges inside the wires; it was the current. So while the scientists were saying that electrons are "particles of electricity," the K-12 textbooks were contradicting them and saying that the motion of the electrons was really the electricity. Both can't be right! And to make matters worse, "Electricity" was already being used to name all sorts of electrical phenomena. In other words, charges and currents in nerve cells are "bioelectricity", while charges and currents in the earth are "geoelectricity," and charges and currents in combed hair are "triboelectricity." Knocking rocks together creates piezoelectricity, and our contracting muscles use myoelectricity. Does this mean that there are many different kinds of electrons? Or many kinds of electrical energy? Of course not. Bio, geo, tribo-electricity are different subject headings in science books. They are neither substances nor energies; they are more like "weather" or "optics." They're different kinds of electrical occurrences, not different kinds of "electricity."

Today when unwary teachers try to understand "electricity", they encounter this morass of contradictions. Often they throw up their hands in frustration and say: "Electricity is really just a kind of event."

This is wrong too!

<Grin>

Pretending that electricity is an event doesn't solve the problem, instead it makes it worse. Teachers are trying to add yet another definition of "Electricity" to the growing list!

The truth is that the word "Electricity" has many contradictory meanings and so the word itself has become meaningless. Electricity is not an event. Neither is it energy, or electrons, or electron motion. Electricity is just a big mistake, but a mistake that slowly crept up on everyone. We never realized it was happening. As long as we keep trying to figure out what "electricity" really is, we'll keep spreading the confusion. The only honest move is to stop hiding the problem. Stop the coverup. We should perform an act of painful honesty, and admit that we've been accidentally misleading generation after generation of students by teaching them about the wonderful substance/occurrence/energy called "electricity" ...which doesn't really exist.


DURING A CURRENT, ELECTRONS IN WIRES START JUMPING FROM ATOM TO ATOM? Wrong.

When individual atoms of copper are brought together to form a bulk metal material, something unexpected happens. The outer electron of each copper atom leaves its parent atom. Rather than orbiting single atoms, the outer electrons all begin "orbiting" around and among ALL the atoms in the metal. Essentially the metal's electrons are "jumping" from atom to atom all the time, even when there is no electric current applied. Physicists call this the "electron sea" or "electron gas" of the metal.

Rather than jumping all the time, what would happen if the electrons only jumped between atoms during electric currents? Well, their jumping motion gives wires their conductivity. If the electrons jumped less during smaller currents, and they stopped jumping during zero current, then the metal conductivity wouldn't be constant, and Ohm's law wouldn't apply. Instead, conductivity would decrease as current decreased. (We know that this doesn't happen.) And if the electrons stopped jumping entirely, the metal would become an insulator. Does copper become insulating during low or zero current? Of course not. So, it's wrong to believe that electric current causes electrons to start hopping. Instead, their constant built-in "hopping" gives conductors a particular value of conductivity. The constant hopping also makes the population of electrons behave like a movable fluid. That's what makes wires so wonderful: they act like pre-filled pipes. They're filled with "liquid electrons." (And when Ben Franklin thought that electricity was a fluid, he really wasn't that wrong.)

In solid metals, not all of the electrons become "loose" and begin wandering. Most are held back by their atoms, and they remain attached to the individual metal atoms. Only the outer electron(s) become part of the "electron sea" of the metal. Different types of metal donate different numbers of electrons to the sea: in some metals, each atom only loses one electron, while in other metals two or more become free. All metals are composed of a mixture of opposite charges: a solid grid of positively-charged atoms which are immersed in a sea of movable electrons. The "electron sea" of metals gives them their characteristics: most other materials will shatter, but metals stay bent because the electrons fill in the gaps. Also, the electron sea is visible to human eyes: it has a silvery color because movable electrons reflect light very strongly. When there is an electric current in a wire, it is these movable electrons which flow. These electrons are not stuck to individual metal atoms in the first place, so they do not need to "jump" during an electric current.

The orbiting motion of the metal's "liquid" electrons takes place at high speed. Doesn't this mean that charge moves fast in wire? No, because this motion has no average direction and is similar to the random thermal vibrations of a gas. It's like a vibration, and it's happening all the time, even when the "wind speed" inside the wire is zero. For this reason we normally ignore the wandering motion of individual electrons, just as we ignore the fast vibration of water molecules when we talk about the speed of a river. And air molecules keep bouncing around fast even when there is no wind at all. The velocity of charges flowing during electric currents is the average "drift velocity," and when the current falls to zero and the charges aren't flowing, they're still wiggling randomly around at very high velocity.

Note that the "Jumping Electrons" misconception in K-12 textbooks is connected with the "Incorrect Definition of Conductor" misconception. The books wrongly treat conductors as hollow pipes for "Current." In truth, conductors are materials which contain movable electricity. If conductors are like plumbing pipes, then these pipes are already pre-filled with water.

MORE PAGES HERE:

 


THE "ELECTRICITY" INSIDE OF WIRES MOVES AT THE SPEED OF LIGHT? Wrong.

In metals, electric current is a flow of electrons. Many books claim that these electrons flow at the speed of light. This is incorrect. Electrons in an electric current actually flow quite slowly; at speeds on the order of centimeters per minute. And in AC circuits the electrons don't really "flow" much at all, instead they sit in place and vibrate. It's the energy in the circuit which flows fast, not the electrons.

Metals are always full of movable electrons. In a simple circuit, all of the wires are totally packed full of electrons all the time. And when a battery or generator pumps the electrons at one point in the circuit, electrons in the entire loop of the circuit are forced to flow, and energy spreads almost instantly throughout the entire circuit. This happens even though the electrons move very slowly.

Electric circuits as drive-belts

To aid your understanding, imagine a large wheel. If you give it a spin, the entire wheel moves as a unit, and this is how you transmit mechanical energy almost instantly to all parts of the wheel's rim. The motion starts at the spot where your hand touches the wheel, and it spreads rapidly from there. But notice that the wheel itself didn't move very fast. The material of the wheel is like the electrons in a wire. Electrical energy is like the "jerk," the mechanical energy-wave which you sent to all parts of the wheel when you gave it a spin. Mechanical energy moves incredibly quickly to all parts of the wheel, but the wheel's atoms didn't have to travel rapidly in order for this to happen.

MORE about electron speed.

Electric circuits as air-filled tubes

Here's another way to understand the problem. Think of sound waves. When we talk, do our vocal cords spew out air molecules? No. Do these molecules zoom out of our mouths at 720 MPH, fly across the empty room, then crash into waiting eardrums? NO! Air molecules are not sound waves. Air molecules do not travel along with sound waves. It's the sound waves that move quickly, not the air molecules. In reality the air barely moves at all, instead the air vibrates back and forth while the sound waves race through the air. The air is the "medium," and sound is a wave which travels through that medium. The same is true of AC electric circuits: the wires are already full of electrons just as the room is already full of air. The electrons in the wire are the "medium" for waves. And when the electrical energy flies along the wires at the speed of light, the electrons do not follow it. Instead the electrons sit in one place and vibrate.

Many people have questioned whether it's really so wrong to teach our kids that electrons move at the speed of light. Well, ask yourself whether it's wrong to teach that sound and wind are the same thing. Is it alright to teach kids that sound is just air molecules which fly through empty space at 720 mph? Obviously it's terribly wrong. The world isn't in a vacuum, and our mouths aren't launching streams of 700MPH molecules. People will have almost no understanding of sound at all if they think that sound is made of fast-moving molecules rather than waves in a medium. To grasp sound, we need to know that sound is a kind of wave. If we don't, then we have a learning barrier, and our understanding of sound stops dead. And kids will have almost no understanding of electricity at all if they think that electrons fly through empty hollow wires at the speed of light. We need to know that electrical energy is a kind of wave. If we don't know this, then we have a very serious learning barrier. Take extra time to get rid of this barrier. Only then can learning continue.

Other articles


THE ELECTRIC ENERGY IN A CIRCUIT FLOWS IN A CIRCLE? Wrong.

When you connect a light bulb to a battery, Electrical Energy moves from the battery to the bulb. This is a one-way flow. The battery loses energy and the bulb gains it. Then the energy received by the bulb is turned into light.

If this phenomenon is examined in great detail, we find that electrical energy is composed of waves traveling along columns of electrons inside the wires, and the energy itself is contained in electromagnetic fields connected to those electrons. We find that it travels as wave energy, that it exists only outside of the wires, and most importantly, that it travels one way along both wires on its trip from the battery to the bulb. The electrical energy did not travel in a circle. So, when you plug a lamp into a wall socket, you shouldn't imagine that the AC energy is a mysterious invisible entity traveling back and forth inside the wires. Instead you should think of AC energy as a mysterious invisible flow that comes out of the outlet, runs along the outside of both wires of the lampcord, then it dives into the filament of the light bulb. Your electric company is sending out long narrow "sausages" made of electrical energy. The wires are guiding them, and your appliances are absorbing them.


THE TWO KINDS OF ELECTRICITY ARE "STATIC" AND "CURRENT?" Nope.

Static and Current are two ways in which electrical charges can behave. If we said that Electrical Science is divided into two fields of research called Electrostatics and Electrodynamics, we'd be correct.

But aren't there several different kinds of electricity? No. We've known this since Michael Faraday first presented a study in 1833 ( published in 1839) where he concluded that all the different "forms" of electricity had an identical cause. For the title of this publication he chose... "Identity of Electricities." See vol.1, p360 in his book "Experimental Researches in Electricity".

Faraday examined the following five situations:
  1. Voltaic piles (Current Electricity)
  2. Electrostatic generators and frictional charge (Static Electricity)
  3. Coils and magnetic induction (Current Electricity)
  4. Thermoelectricity (Seebeck effect)
  5. Bioelectricity ("torpedo" ray and Electric Eel)
Faraday concluded that all these were simply situations where the charge and the current had different values. We believe the same today: for example, so called "static electricity" involves high voltage at little or no current. Are batteries a source of "current electricity?" Well, stack up enough batteries in series, and the ones on the end will attract lint, produce corona discharges, and cause hair to rise. And hook up enough VandeGraaff machines in parallel, and you can light up a standard fluorescent tube.

But suppose you don't trust authorities like Faraday? Well, instead let's examine the situation itself. First, please realize that the study of water is divided into Hydrostatics and Hydrodynamics, yet we don't go around constantly discussing a special kind of water called "static water," nor do we think that "current water" is a kind of invisible energy. Water can move, and water can be pressurized, but it's still just one kind of water. The same applies to electric charges.

For those among us who insist that "Static electricity" and "Current electricity" are two separate kinds of electricity, then please explain the following. Whenever positive and negative charges are forced to separate as they flow along a pair of wires, then those wires becomes electrostatically charged... but the charges are not static. Instead they're flowing along. Yet even while the charges flow, those wires will cause hair to rise, and they can attract bits of fur or lint... all while the so-called "static electricity" is moving along as an electric current. How can "static electricity" be dynamic? Does this make your brain ache?

The solution is simple: just realize that "static electricity" is a misnomer. "Static" is actually composed of forcibly *separated* opposite charges, and if those separated charges should flow along, they still behave as "static electricity," regardless of their motion. The key is the separation of the charges...while their "static-ness" is not important. For this reason, charges can behave as "static electricity" and "current electricity" both at the same time. This is not so terrible, since water behaves in a similar way: water can be pressurized, and it can flow at the same time. A flow of high-pressure water simultaneously falls under the two subjects of "hydrostatics" and "hydrodynamics." Fortunately we don't confuse students by calling high-pressure water by the name "static water." Maybe we should change the name of "Static electricity" to something sensible, like "charge imbalance", or "pressurized electricity." It would end a lot of confusion.

So to sum up... charges can flow, and opposite charges can be forced to separate and become un-cancelled, but this doesn't mean that "flowing electricity" is a different kind of charge than "separated electricity." Separation and flow are two electrical behaviors, they are not two "kinds of electricity."

More about this: WHAT IS ELECTRICITY?


CURRENT ELECTRICITY IS THE OPPOSITE OF STATIC ELECTRICITY? Wrong.

It was Michael Faraday who discovered in 1839 that "static electricity" and "current electricity" don't actually exist.

Faraday showed that electric eels can charge a capacitor or deflect an ammeter. So-called "animal electricity" was not a separate type.

He also showed that rug-scuffing or "frictional" electricity was simply a low current being produced at a high voltage (small I at high Q,) and like the Electric Eel, it could be used to deflect ammeters or even perform electrolysis/electroplating. He did the same thing with volta-piles (batteries,) electrostatic generators, thermocouples, and piezo devices. He concluded that there was only one "kind" of electricity, although different sources would produce different values of Q and I, of charge and current. See M. Faraday, 1839 "Experimental Researches in Electricity," Vol 1, p360

But it's 170 years later, and while grade school textbooks aren't pretending that "Bioelectricity" is a separate type, they're still insisting that "static" and "current" are the two opposite "forms of electricity." No, wrong. It's true that scuffing on a rug will produce thousands of volts at currents of tens of microamperes. Waving a magnet near a copper plate can induce currents of thousands of amperes at voltages of tens of microvolts. As Faraday found, these aren't two opposite "forms of electricity."

Some more detail... so-called "Static" electricity appears whenever the negative charges within matter are separated and held apart from the positive charges. On the other hand, electric currents appear whenever the negative charges within matter are made to flow through the positive charges (or when positives flows through the negatives.) "Static" and "Current" are two entirely independent events. They are not opposites, and they can both appear at the same time.

"Static" is actually a separation; it is a stretching-apart, and it really has little to do with anything remaining static or stationary.

"Current" is a flowing motion. It has little to do with the separation of opposite charges.

"Static electricity" was misnamed. It's not a separate kind of electricity, so really it should be called "charge separation" or maybe "high-tension electricity." It's "stretched" or "pressurized" electricity. Since tension is not the opposite of motion, Static is not the opposite of Current. And although electric current really exists and electric charge really exists, there is no such energy or material called "current electricity" or "static electricity." See MORE on this topic.

 

THE STUFF THAT FLOWS THROUGH WIRES IS CALLED 'ELECTRIC CURRENT'? Horribly misleading!

Most K-12 textbooks discuss a substance or energy called "current". They constantly talk about "flows of current." However, here's a pointed question: what flows in rivers? Is it water, or is it "current?" If I fill a bucket from the faucet, is my bucket full of "current?" No! Water moves, water flows in pipes, not "current." A flow of water is a correct phrase, while "a flow of current" is not. The same idea applies to electricity: electric current is a flow of a substance, but the name of the substance is not "current."

Since a current is a flow of charge, the common expression "flow of current" should be avoided, since literally it means "flow of flow of charge."

              - Modern College Physics: Sears, Wehr, & Zemansky
(Find similar in Serway's book COLLEGE PHYSICS)
Another question: what would be the consequences if the English language had no word for "water", but instead we called it "current"? What if everyone actually believed that rivers and plumbing pipes were full of "current?" What if we believed that "current" flows in rivers... yet we had no name for water? Yes it would be bizarre, but here's the thing: wouldn't people tend to acquire many serious misconceptions about the nature of water? For example, we might imagine that water vanishes whenever it stops flowing. After all, "current" goes away when the river is halted. And when we turn off a faucet, we'd imagine that the pipes became empty. After all, a halted current is... nothing! A glass of water would seem very confusing, since the glass would be full of some weird kind of "stationary current." Students would ask all kinds of questions about the confusing nature of "current," and their teachers wouldn't know how to answer.

As far as elementary textbooks are concerned, we have no name for the thing which flows inside of wires. Yes it's true that this entity, whenever it is flowing, is properly called "an electrical current." But when the stuff *stops* flowing, what do we call it? It's still inside the wire of course. But it's not moving anymore. Refer to advanced physics texts, and there we'll find its correct name: Charge. An electric current is a flow of charge. Yet the K-6 books never mention this. Instead they say that "current flows." They say it over and over. And over! And any students are very lucky if they avoid picking up the wrong idea that the charges vanish when the flow is halted. (Does the water in a pipe suddenly evaporate when you halt its flow? No, and neither do the flowing charges within a metal wire. The "current" is gone, but the charges just stopped in place.)

Worse, most books say that "current electricity" flows inside wires. They don't mention charge-flow. To this I ask, "Is there a special kind of water called 'Current Water?'" The answer obviously is NO. This same answer applies to electricity: there is no special "electricity" called Current Electricity. Electricity can flow and electricity can stop, and a flow of electricity (or charge) is called an Electric Current, but there is no such thing as Current Electricity.

Here's a useful hint for authors: to verify accuracy in your articles, temporarily remove the word "current" and replace it everywhere with the term "charge flow." Then check to see if your sentences still make sense. If a sentence states that charge-flow is a form of energy, then you've found a mistake. If it states that charge-flow flows, or talks about flows of charge-flow then that particular sentence is probably confusing to students, and is teaching them to believe in an impossible and contradictory substance called "current electricity."


ELECTRIC CURRENT IS A FLOW OF ENERGY? Wrong.

Electric current is not a flow of energy; it's a flow of charge. Charge and energy are two very different things. To separate them in your mind, see this list of differences.

An electric current is a flowing motion of charged particles, and the particles do not carry energy along with them as they move. A current is defined as a flow of charge by I=Q/T; amperes are coulombs of charge flowing per unit time. The term "Electric Current" means the same thing as "charge flow." Electric current is a very slow flow of charges, while energy flows fast. Also, during AC alternating current the charges move slightly back and forth while the energy moves rapidly forward.

Electric energy is quite different than charge. The energy traveling across an electric current is made up of waves in electromagnetic fields and it moves VERY rapidly. Electric energy moves at a completely different speed than electric current, and obviously they are two different things flowing in wires at the same time. Unless we realize that two different things are flowing, we won't understand how circuits work. Indeed, if we believe in a single flowing "electricity," we will have little grasp of basic electrical science.

In an electric circuit, the path of the electric charges is circular, while the path of the energy is not. A battery can send electric energy to a light bulb, and the bulb changes electrical energy into light. The energy does not flow back to the battery again. At the same time, the electric current is different; it is a very slow circular flow, and the electric charges flow through the light bulb filament and all of them flow back out again. They return to the battery.

Electric energy can even flow in a direction opposite to that of the electric current. In a single wire, electric energy can move continuously forward while the direction of the electric current is slowly backwards. In AC circuits the energy flows continuously forward while the charges are alternating back and forth at high frequency. The charges wiggle, while the energy flows forward; electric current is not energy flow.

Here's one way to clarify the muddled concepts: if electric current is like wind, then electrical energy is like some sound waves, and the electrons are like the molecules of the air. For example, sound can travel through a pipe if the pipe is full of air molecules, and electrical energy can flow along a wire because the wire is full of movable charges. Sound moves much faster than wind, correct? And electrical energy moves much faster than electric current for much the same reason. Air in a pipe can flow fast or slow, while sound waves always move at the same very high speed. Charges in a wire can flow fast or slow, while electrical energy always flows along the wire at a single incredibly high speed. Whenever sound is flowing through a pipe, the air molecules in that pipe are vibrating back and forth. And when waves of AC electrical energy are flowing along a wire, the electrons in that wire are vibrating back and forth 60 times per second.

Suppose that we were all taught that sound and wind are the same thing? This would prevent us from understanding both wind and sound. We'd be forever wondering why air molecules DON'T travel along at 720MPH, while sound does travel this fast. Where electricity is concerned, K-6 textbooks teach us that "sound" and "wind" are the same. They wrongly teach that electric currents are a flow of energy, as if wind were really sound. It completely prevents us from understanding both electric current and energy flow.

Be careful, since my description of the above pipes are just an analogy, and sound waves aren't *exactly* like electrical energy. For example, sound can flow inside an air-filled tube, while electrical energy always flows in the space outside of the wires, and does not travel along within the metal wires. However, electrical energy is coupled with compression waves in the electrons of the wire. Electron-waves travel inside the wires, yet the energy they carry is in the invisible fields surrounding the wires.

Is it actually important for people to understand that wind is not sound? Obviously yes! School books would cause harm if they taught everyone that wind is sound. And if we want to understand circuits, we need a clear view of electric charge-flow, and of electric energy flow. Students need to be totally certain that they are two different things, and our school textbooks teach us the exact opposite.

 

ELECTRICITY LEAVES ONE BATTERY PLATE, THEN RETURNS TO THE OTHER? Not quite.

In a simple circuit, the actual path of electric current is through the battery. Some books imply (or even state outright) that whenever a battery is connected in a complete circuit, the charges only flow in the wires. They say that one battery plate is the source of electrons, the other plate collects "used" electrons, and no charges flow in the chemicals between the battery plates. This is wrong. These books often contain a diagram of a battery, wires, and a light bulb. The diagram shows the current in the wires, but charges aren't shown going through the battery. This is wrong. The electrolyte is a conductor. There is a large current flowing between the battery plates.

In any simple electric circuit, the path of the electric current is a complete circle. It's like a drive belt, and it has no starting point. It goes through all parts of the circuit including the battery, and including the battery's liquid electrolyte. Between the plates of any battery we find a highly conductive liquid. Sometimes its an acid (as with car batteries.) Sometimes it's alkaline, and sometimes it's salt water. (But how can a battery even work, if the plates are connected together with conductive fluid? That's a separate topic.)

No charges are continuously building up in the battery plates. There's only a small build-up to "pressurize" the terminals and create the flow. And if there's one Ampere in the wires connected to the battery, then there's also a 1-Amp flow of charge in the electrolyte between the two plates. Where does this charge come from? Go down to this section. A battery does not supply charges, it merely pumps them. Whenever electric charge flows into one terminal of a battery, an equal amount of charge must flow through the battery and back out through the other terminal. In a simple DC battery/bulb circuit, the charges are forever flowing around and around the circuit, going through the bulb filament, but also going through the battery. The battery is a charge pump.


ELECTRIC ENERGY IS CARRIED BY INDIVIDUAL ELECTRONS? Wrong.

Some books teach that, in a simple battery/bulb circuit, each electron carries energy to the bulb, deposits its energy in the hot filament, and then returns to the battery where it's re-filled with energy. This is wrong. Some books give an analogy with a circular track full of freight cars waiting to be filled with coal. This picture is wrong too. The energy in electric circuits is not carried by individual electrons. Instead the electrons flow very slowly while the electrical energy flows rapidly along the columns of electrons. In AC circuits the electrons don't flow forward at all, instead they vibrate slightly, even though the energy flows rapidly from generator to appliance. As with rotating shafts and drive belts, the energy is carried by the circuit as a whole, not by the individual particles in the system.

Here's an analogy which may help explain it:

Imagine a wheel that's free to spin. For example, turn a bicycle upside-down in your mind. Give the front tire a spin. When you spin the wheel, your hand injects energy into the entire wheel all at once. Now put your hand lightly against some part of the tire so the spinning wheel is slowed and stopped by friction. Your hand gets hot. Your hand extracts energy from the entire wheel, all at once, and the whole wheel slows down. Finally perform both tasks at once: rub one hand lightly against the tire, while you use your other hand to keep the wheel spinning.
Would it be right to tell students that the "Power" hand fills each rubber molecule with energy, that the molecules travel to the "Friction" hand and dump their energy? Then the empty molecules return to the "power" hand and get refilled? No, of course not! If this were true then the energy would be forced to travel only as fast as the rubber. Your "friction" hand wouldn't experience any friction until those magically energized rubber molecules made their way around the rim. Part of the wheel would be spinning while part would be de-energized and unmoving, and this would be really a strange sight to see!

Instead, one hand spins the wheel and fills the whole thing with kinetic energy... and the rubbing hand sucks the energy back out at the same time. The wheel rotates, and energy flows almost instantly across the wheel, going from the "spin" hand to the "friction" hand.

A flashlight circuit is like our bicycle wheel. The electrons in the copper wire circuit are like the rim of the wheel. Electrons are like an invisible drive-belt hidden inside the wires. The battery causes all the electrons in the loop of wire to begin moving. In this way it injects energy into the whole circuit all at once, just like a hand that spins a bicycle wheel. As soon as the battery moves the circuit's electrons, the distant lightbulb lights up. The electrons moving into the bulb's filament are exactly the same as the ones moving out; the bulb doesn't change them or extract stored energy from them. (Did your hand do anything to change the rubber molecules as it rubbed on the bicycle wheel? No, it just slowed down the entire wheel.) A hand can extract energy from an entire bicycle wheel instantly, and the hand heats up by friction. Same thing with the bulb, it slows down ALL the electrons throughout the entire circuit, and in this way the bulb extracts energy from the whole circuit as it lights up.

In other words, electric circuits are drive belts, and all wires are always full of movable electrons.

In discussing the "freight cars" misconception with teachers, I find that they see nothing wrong with teaching the wrong picture to their students! After all, the kids instantly grasp the "freight cars with coal" story since it's very visible and it offers a sensible explanation. What more can we ask? Yet there is a serious problem here: electrons flow slowly, and in AC circuits they don't flow at all, instead they wiggle. If the freight cars only vibrate, and never flow forwards, how the heck does any coal get to the other side of the circle? (How does AC actually work?) There's no answer. Students will be trapped.

In order to really understand electric circuits in the more advanced classes, a student must unlearn the seductive freight-cars analogy. "Unlearning" rarely happens, and so this wrong analogy can form a learning barrier which can forever prevent any further progress. It freezes their understanding of electricity at the elementary-school level. Yes, if those particular kids will never have any need to understand how electricity really works, then the freight-cars analogy is fine. The kids can memorize it, teachers can test them for it, and everybody is happy. "Learning" has occurred (rather than actual learning.) But if the kids grow up to become scientists and engineers and technical people, then the freight-cars analogy causes them harm. Unfortunately, it only causes future harm, so the grade-school educators never encounter the negative effects of the misconceptions they've installed in their kids' minds.

OK, what if you were using the "freight cars w/coal" analogy, but you also had to explain how "AC" works? In that case the freight cars are moving back and forth but not progressing forwards. How can they deliver their coal to the far end of the track? I suspect that teachers encounter this problem, but rather than recognizing that "freight cars" is a misconception, they instead pile another misconception: the wrong idea that electrons in wires flow at the speed of light. After all, if the coal-filled freight cars traveled instantly to the far end of the track, then dumped their coal, then traveled instantly back, that would be alternating current. Right?!! Wrong, because electric currents are actually very slow flows of charges. During AC those "freight cars" only wiggle back and forth a few inches on their tracks.

The bicycle-wheel analogy has no problem explaining AC. Just wiggle the bicycle wheel back and forth instead of spinning it continuously. The wiggling wheel will rub upon the distant unmoving "friction" hand, and will heat up that hand. Energy can essentially travel instantly across the bicycle wheel, even though the wheel itself rotates slowly. Energy can travel instantly between the two hands, even if the wheel moves back and forth instead of spinning. What determines the direction of this energy travel? It's simple. If one hand spins the wheel, it throws energy out into the wheel, and if another hand rubs on the tire, it extracts energy from the wheel. Notice that the energy doesn't care about the wheel's rotation. The energy flows one-way, from one hand to the other, even if the wheel reverses direction, and even if the wheel vibrates back and forth rather than continuously turning. And most important: both paths through the tire are sending energy in one direction: towards the frictional load. In one side, the energy is going backwards against the direction of current.

The "filled freightcars" analogy only becomes seductively appropriate when used to explain Direct Current. However, when explaining Alternating Current, the analogy breaks down completely. Each freight car wiggles back and forth, so how can those energy-filled buckets move from the "battery" to the "light bulb?" They cannot. The analogy doesn't work, and students who have learned the analogy will find it impossible to understand AC. In grade-school they were taught about batteries and bulbs, but never about power plants and AC electric heaters. Again, this is fine if the kids have no hopes of entering any kind of technical career; if their science learning will cease after fifth grade...

How about another analogy about this analogy (grin!) Sound waves are much like electrical energy in circuits. So, how do sound waves work? Ask yourself this: would it be OK to teach kids that your vocal chords place energy into air molecules, then each air molecule zooms out of your mouth at 720MPH, then flies across the room almost instantly, then eventually crashes into the ears of distant listeners? That's silly. There's no supersonic wind coming out of our mouths. I would think that any author who use this kind of explanation should be ashamed. Yes, the explanation "works", and it is easy for the kids to grasp. But it is totally wrong: sound is carried by waves in the air, not by individual air molecules launched at immense velocities out of your mouth. And any kid who believes this "launched molecules" sound-explanation will eventually have terrible difficulties should they ever have need to understand how sound really works.

All of this is an analogy for wires and circuits: electrical energy is wave energy; electrical energy moves along the columns of electrons like sound waves move through the air, and when electrical energy flows across a circuit, the electrons DON'T flow along with it. Electrons are a wave-medium, and all electrical energy is wave-energy. (Don't forget that electrical energy has another name: electromagnetism.) Yank on a rope, and the rope moves towards you, but the "jerk" flies rapidly in the other direction along the rope. The "jerk" is the energy. It travels as a wave through a medium. But this means ...electric circuits have a rope inside! Exactly right. That's why you need a complete circuit, since if there is a blockage anywhere in the, ahem, "circuit," the ring of invisible electricity-rope cannot move.


BATTERIES STORE CHARGE, AND THIS CHARGE FLOWS IN WIRES? No.

The word "charge" has more than one meaning, and the meanings contradict each other. The "charge" in a battery is energy (chemical energy), while the "charge" that flows inside wires is part of matter, it is electron particles. And those wires, even though full of charge... are neutral and uncharged! The term "charge" refers to several different things: to net-charge, to quantities of charged particles, and to "charges" of energy. If you are not very careful while using the word "charge" in teaching, you might be spreading misconceptions.

For example, even when metals are totally neutral, they contain vast quantities of movable electrons, immense charge. So, should we say that they contain zero charge because they are neutral? Or, should we say that they contain a very large amount of electric charge, because they are filled with many Coulombs of electrons? Don't answer yet, because your answer might be inconsistent with how we describe capacitors (further below.)

Another: if I place an electron and a proton together, do I have twice as much charge as before, or do I have a neutral hydrogen atom with no charge at all? What I do have is confusion. Misuse of "charge" makes descriptions of electric circuits seem complex and abstract, when the explanations are really just wrong.

Another: electric currents in wires are actually a motion of "neutralized" charge, where every electron has a proton nearby. If we teach our kids that a wire is uncharged, and we also teach that electric current is a flow of charge, how can anyone make sense of a situation where a wire has no charge at all, yet contains an enormous flow of charge? We could say "Oh, but most electric currents are usually a flow of Uncharged Charge." WHAT? What would a student make of that statement? Can you see the problems that arise because of the word "charge?"

Another one: as you "charge" a battery, you cause an electric current to appear in the electrolyte, and this motion of electric charges causes chemical reactions to occur upon the surfaces of the battery's plates. Chemical "fuel" accumulates, but charge does not: the charges flow into (or out of) the surfaces of the plates and do not accumulate there. (The path for electric current is through the battery. Through, and back out again.) A "charge" of chemical energy is stored in the battery, but electrical charge is not. And when a battery is being "discharged", it's chemical fuel drives a process which pumps charge through the battery. During discharge the battery's fuel will eventually be exhausted, but the total electric charge within the battery will never decrease!

Here's a way to imagine the process: a battery is like a spring-driven "wind up" water pump. Send water backwards through this pump, and you wind up the internal spring. Then, provide a pathway between the inlet and the outlet of the pump, and the spring-motor will pump the water in a circle. But now think for a moment: the water is the charge, yet our wind-up pump does not store water! When we "charge" our wind-up pump, we send the charge (water) through the pump, and this stores energy by winding up the spring. Same with a battery: to "charge" a battery, we send electrical charges through the battery and back out again. This causes the chemicals on the battery plates to store energy, just like winding up the spring in our spring-powered water pump. See how "charging" and "charges" can create a horrible mess of misunderstandings? When this mess gets into the K-12 textbooks, and educators start teaching it to kids, the kids end up believing that Electricity is far too complicated for them to understand. Yet the fault does not lie with the students!!!!

Another one: if you "charge" a capacitor, you move charges from one plate to the other, and the number of charges within the device as a whole does not change. Or from an engineer's perspective, you drive charge through the capacitor, which causes potential across the plates to rise, and the rate of rise is proportional to the charge-flow. But capacitors have exactly the same total charge within them whether they are "charged" or not! Whenever we take an electron from one plate, we put an electron onto the other plate. Whenever we speak of "charging" capacitors, we've suddenly stopped talking about charge, and started talking about electrical energy. A "charged" capacitor has quite a bit more energy than an "uncharged" capacitor (but exactly the same net-charge, and the same quantity of + and - particles inside.) This basic concept reinforces the idea of charge-conservation, and is very important in understanding simple circuitry, yet it is rarely taught. The misleading term "charge" stands in the way of understanding. I suspect that students are not the only ones being misled. Many teachers misunderstand simple physics, and they truely believe that the purpose of a capacitor is to store electric charge.

Think like this: both capacitors and inductors (coils) store energy. They store energy in the form of fields. Coils don't store any electric charge. Capacitors don't store any electric charge. Yet electric charge is the medium of energy-storage in both coils and capacitors. In capacitors, energy is stored in the form of "stretched charge", or electric potential energy, where positive and negative charged particles are pulled away from each other and held separated. Coils store energy in the form of relative moving charge which contains a sort of electrical kinetic energy. However, we don't put any charge into a capacitor when we "charge" it, any more than we put charge into a superconductive ring-inductor when we give the ring a "charge" of electromagnetic energy.


"STATIC ELECTRICITY" (CONTACT ELECTRIFICATION) IS CAUSED BY FRICTION? Wrong.

"Static electricity" is not caused by friction. It appears when two dissimilar insulating materials are placed into intimate contact and then separated. All that's required is the touching. For example, when adhesive tape is placed on an insulating surface and then peeled off, both the tape and the surface will become electrified. No rubbing was required. Or when a plastic wheel rolls across a rubber surface, both the surface and the wheel become electrified. Intimate contact is sufficient, and no rubbing is needed. In a VandeGraaff generator the rubber belt pushes against the plastic roller, then peels away again. Of course if one of the two materials is rough or fibrous, then it does not give a very large footprint of contact area. In this case the process of rubbing one material upon another can greatly increase the total contact area. And the heating of the fibers can make the materials even more electrically "dissimilar", which aids the charge-separation process. But this rubbing is not the cause of the electrification.

"STATIC ELECTRICITY" IS A BUILDUP OF ELECTRONS? Wrong.

It is not a buildup of anything, it is an imbalance between quantities of positive and negative particles already present.

During contact-electrification it is usually only the negative electrons which are moved. As negative particles are pulled away from the positive particles, equal and opposite areas of imbalance are created. In one place you'll have more protons than electrons, and this spot will have an overall positive charge. Elsewhere you'll have more electrons than protons, for an overall negative charge. You've not caused a "buildup", you've caused an imbalance, an un-canceling, a separation. In fact, the scientific term for static electrification is charge separation. The law of Conservation of Electric Charge requires that every time you create a region of negative charge, you must also create a region of positive charge. In other words you must create a separation of opposite charges. If you want to call it a "buildup of electrons", then you also need to call it a "buildup of protons," since you can't have one without the other.


"STATIC ELECTRICITY" IS ELECTRICITY WHICH IS STATIC? Wrong.

"Static electricity" exists whenever there are unequal amounts of positive and negative charged particles present. It doesn't matter whether the region of imbalance is flowing or whether it is still. Only the imbalance is important, not the "staticness." To say otherwise can cause several sorts of confusion.

All solid objects contain vast quantities of positive and negative particles whether the objects are electrified or not. When these quantities are not exactly equal and there is a tiny bit more positive than negative (or vice versa), we say that the object is "electrified" or "charged," and that "static electricity" exists. When the quantities are equal, we say the object is "neutral" or "uncharged." "Charged" and "uncharged" depends on the sum of opposite quantities. Since "static electricity" is actually an imbalance in the quantities of positive and negative, it is wrong to believe that the phenomenon has anything to do with lack of motion, with being "static." In fact, "static electricity" can easily be made to *move* along conductive surfaces. When this happens, it continues to display all it's expected characteristics as it flows, so it does not stop being "static electricity" while it moves along very non-statically! In a high voltage electric circuit, the wires can attract lint, raise hair, etc., even though there is a large current in the wires and all the charges are flowing (and none of the electricity is "static.") And last, when any electric circuit is broken and the charges stop flowing, they do *not* turn into "static electricity" and begin attracting lint, etc. A disconnected wire contains charges which are not moving (they are static,) yet it contains no "static electricity!"

To sort out this craziness, simply remember that "static electricity" is not a quantity of unmoving charged particles, and "static electricity" has nothing to do with unmoving-ness. If you believe that "static" and "current" are opposite types of "electricity," you will forever be hopelessly confused about electricity in general.


ELECTRIC POWER FLOWS FROM GENERATOR TO CONSUMER? Wrong.

Electric power cannot be made to flow. Power is defined as "flow of energy." Saying that power "flows" is silly. It's as silly as saying that the stuff in a moving river is named "current" rather than named "water." Water is real, water can flow, flows of water are called currents, but we should never make the mistake of believing that water's motion is a type of substance. Talking of "current" which "flows" confuses everyone. The issue with energy is similar. Electrical energy is real, it is sort of like a stuff, and it can flow along. When electric energy flows, the flow is called "electric power." But electric power has no existence of its own. Electric power is the flow rate of another thing; electric power is an energy current. Energy flows, but power never does, just as water flows but "water current" never does.

The above issue affects the concepts behind the units of electrical measurement. Energy can be measured in Joules or Ergs. The rate of flow of energy is called Joules per second. For convenience, we give the name "power" to this Joule/sec rate of flow, and we measure it in terms of Watts. This makes for convenient calculations. Yet Watts have no physical, substance-like existence. The Joule is the fundamental unit, and the Watt is a unit of convenience which means "joule per second."

I believe that it is a good idea to teach only the term "Joule" in early grades, to entirely avoid the "watt" concept. Call power by the proper name "joules per second". Only introduce "watts" years later, when the students feel a need for a convenient way to state the "joules per second" concept. Unfortunately many grade-school textbooks do the reverse, they keep the seemingly-complex "Joule" away from the kids, while spreading the "watt" concept far and wide! Later they try to explain that joules are simply watt-seconds! (That's watts times seconds, not watts per second.)

If you aren't quite sure that you understand watt-seconds, stop thinking backwards and think like this: Joules are real, a flow of Joules is measured in Joules per second, and "Watts" should not interfere with these basic ideas.


PROTONS CANNOT FLOW? Not so.

In metals, yes you're right, the protons of the metal atoms cannot flow along. But never forget that metals aren't the only type of conductor. So, where would we encounter some flowing protons?

First, note that proton currents are typically called something different. From a chemistry viewpoint, a single proton is really nothing but a hydrogen atom which is lacking any electrons. So, the other common name for proton is H+, or "positive Hydrogen ion!" Where do we find electric currents made of + hydrogen ions? Car batteries, of course. When H2SO4 is added to water, it splits up (ionizes) to become the negative SO4 ion plus two H+, two mobile protons.

In acids the electric current is a flow of protons in one direction and negative ions in the other. The protons are much more easily moved, so most of the electric current in acid solutions is from the proton flow.

And besides acids, pure water also is a conductor where the currents are +H ions (protons) and -OH negative ions, each travelling in opposite directions. (Instead of metal wires full of electrons, imagine a long hose full of water. Push one proton into one end, and immediately a proton pops out of the far end! That's proton flow.) Interesting fact: ice that's made from fresh water is not a perfect insulator, instead the OH- ions are immobilized in the ice crystal: it's a proton conductor.

Recently with the rise of green-energy topics we have another common proton conductor. These are called ...proton conductors. They're used in fuel cells. Another name for them is "solid electrolyte." In these types of conductors the charge carriers are protons.

PS Why do chemistry texts call protons by the name "hydrogen ion?" Part of this is just tradition: we knew about hydrogen, and then about atom ionization, long, long before we knew about protons. (Heh, so why don't chemists call Helium by the name "alpha particle?") But also, protons are extremely reactive chemically, and it makes sense to view them as positive charged hydrogen, the first element in the Periodic Table. Also, hydrogen isn't pure protons, it's not a pure isotope. When hydrogen is present, a tiny fraction of a percent is actually deuterium, a proton plus a neutron, and far smaller fraction has two neutrons: tritium. Your water is mostly made of protons and oxygen, but a bit of it is heavy water, and a slight amount is radioactive. To conceal this complexity, we could pretend that it's all protons. Or we could avoid the issue and just call it Hydrogen.

And finally, please notice that all of this goes directly against the common widespread misconception that "all electricity is really electrons."


LIGHT AND RADIO WAVES TRAVEL AT 186,000 MILES PER SECOND? No.

They only travel at 186,000 miles per second while in a perfect vacuum. Light waves travel a bit slower in the air, and they travel lots slower when inside glass. And radio waves move much slower than 186,000 miles/sec when they travel within plastic-insulated coaxial cable. The term "speed of light" is misleading, because the complete term actually reads "speed of light in a vacuum." There actually is no set "speed of light" because light waves and radio waves (and electrical energy) can travel at many different speeds depending on the medium through which the waves propagate.
[NOTE: I receive complaints insisting that the speed of light is always the same, and that glass slows the light waves by atomic absorption and emission. Yet this is a very distorted viewpoint, since it denies the existence of extremely useful mental models called "transparent medium" and "EM wave." Also it wrongly teaches us that matter is not matter, but is really a vacuum. Yet light waves really do travel at various speeds (traveling slowly within glass,) and optical materials really are not a vacuum. The atoms of the glass do not absorb waves, they absorb photons. Our focusing so completely on atom/photon interactions denies the wave nature of light, pretends that E and M fields don't exist, and also entirely misses the existence of macroscopic phenomena such as transparency. It also makes a fundamental mistake: declaring one physics model to be "real," then pretending that this can make other useful models UNreal. But all physics models are merely mental abstractions, they are tools. The photon concept does not eliminate the fields/waves concept. The worshippers of screwdrivers think that hammers are a sort of "inferior screwdriver," and should be abolished? Silly, because hammers really aren't screwdrivers, and screwdrivers are genuinely worthless for certain tasks.



ELECTROMAGNET COILS USE UP ENERGY TO MAKE MAGNETISM? Not right.

Sustaining a magnetic field requires no energy. Coils only require energy to initially create a magnetic field. They also require energy to defeat electrical friction (resistance); to keep the charges from slowing down as they flow in wires. But if the resistance is removed, the magnetic field can exist continuously without any energy input. If electrically frictionless superconductive wire is used, a coil can be momentarily connected to an energy supply to create the field. Afterwards the power supply can be removed and both the current and the magnetic field will continue forever without further energy input.

VOLTAGE

See: What Is Voltage

ELECTRIC CHARGES ONLY FLOW ON THE SURFACES OF WIRES? Wrong.

During a Direct Current in a simple circuit, the flow of charges takes place throughout the whole wire. The flow is not just on the surface. If the level of current is very high, then the wire will become hot, and the current will heat up the inside of the wire as well as its surface. Thin hollow pipes make poor conductors; their electrical resistance is too high. To avoid overheating the metal we should use thick solid bars instead.

There is a persistent 'rumor' that the path for flowing charges is entirely on the surface of metals. This mistaken idea probably comes about through a misunderstanding of the nature of electric charge. After all, when some electric charge is deposited onto a metal object, it distributes itself over the surface of the object. It makes sense that, since charge is only on the surface of metals, a flow of charge must take place only on the surface of metals, right? Wrong. Unfortunately, the word "charge" refers to two different things. When electric charge is placed on a metal object, the added charge is just a drop in the bucket compared to the amount of charge already in the neutral metal. "Uncharged" wires contain an enormous amount of movable charge inside, even though they may be "neutral" and so have zero charge on average. Are you confused yet?

All metals contain enormous amounts of free, movable electrons. During an electric current it's these electrons which flow along. However, each electron is near a proton, and so the metal is said to be "uncharged." In a wire, electric current is a flow of this "uncharged" charge. Weird but true. Now if we were to place extra charge upon a wire, that would be like pouring a teacup into the ocean. The "water level" would rise a tiny bit. The currents inside the "ocean" would be unaffected. Yet extra charges on a wire also create a very noticeable electrical imbalance (they attract lint, deflect electroscopes, make sparks, etc.)

It isn't so strange that we might accidentally assume that the extra surface charges are the metal's only charges. Yet in reality, electric currents happen in the "ocean" of the wire, and the extra "teacup" of unbalanced charge on the surface has little effect on the overall charge-flow. The charge-flow (current) is not just on the surface, and during electric current, the whole "ocean" flows.

A second source of misunderstandings: during high frequency AC, the value of electric current in a conductor is higher at the surface than it is within the bulk of the metal. This is called the "skin effect." It is not very important for thin household wires at 60Hz. Perhaps some people heard about the Skin Effect but did not realize that it only works for very thick wires or for high frequency AC. At extremely high frequencies, only the charges in a thin "skin" on the surface of large wires are the charges which move. For circuits involving high-current and high-frequency such as radio transmitters, it makes sense to use copper pipes as conductors. All the charge-flow is on the surface of the conductors, so use inexpensive hollow conductors. All the heating takes place on the surface, and not deep within the metal. But note well that the thin "skin" of current is a *different* thing than the thin "skin" of excess charge-imbalance found on an electrified wire.


ELECTRIC CHARGES ARE INVISIBLE? No.

Electric charges are easily visible to human eyes, even though their motion is not. "Electricity" is not invisible! Never has been. When you look at a metal wire, you can see the charges of electricity which would flow during electric currents. They are silvery/metallic in color. They give metals their mirrorlike shine. Some metals have other colors as well, brass and copper for instance. Yet in all cases, the "metallic"-looking stuff is the metal's electrons. A dense crowd of electrons looks silvery; "electric fluid" is a silver liquid. And if metals weren't full of movable electrons, they wouldn't look metallic.

During electric currents in metals, the atoms stay still, but the silvery electron-stuff flows slowly along. Unfortunately the human eye cannot see the electric flow. That's part of the reason that "electricity" is so mysterious. Think about it... in an aquarium full of water, you cannot see any water flowing unless there are bubbles or dirt being carried along. And whenever clean water is flowing through a transparent hose, you can't see any flow. Even if the water is flowing very fast, the water-filled hose just looks like an unmoving glass rod. Same with wires: there's no bubbles or dirt being carried along by the electric current, therefore you can't see anything moving. You can see the stuff that flows, just as you can see the water in an aquarium, but you can't see any flowing stuff.

Even if human eyes could see single electrons, we still couldn't see an electrical flow since the current is extremely slow. Electrons in metals typically flow at a few centimeters per hour, even during high currents. That's slower than the minute hand on a clock! Electric currents ooze along like silly-putty flowing across a tilted board.

------

SEEING IMBALANCES OF CHARGE

Here's a somewhat-separate topic. While the metallic-looking sea of charges in a metal is easily seen, imbalances of charge remain invisible. This gets confusing, since many authors call imbalances of charge by the name "charge." They will tell you that charge is invisible, yet they really mean that net-charges or charge-imbalances are invisible.

Wires contain enormous amounts of movable negative charge in the form of electrons, but they also contain positive charge in the form of protons within the metal atoms. If the number of protons and electrons are equal, don't they cancel out? Doesn't that mean that no charge exists? No. It means that no imbalance of charge exists. It means that, while the wire is full of mobile charge, the net-charge remains zero.

An "uncharged" wire is still full of charge, it still contains positive and negative charge in huge but equal quantities. The word "uncharged" doesn't mean "without charge," instead it means "without charge-imbalance." Yet even if there are more electrons than protons, or fewer electrons than protons, this imbalance is invisible. It's invisible because the greatest difference attainable is yet incredibly tiny when compared to the amount of charge that's already there. If an object is highly charged; even charged up to millions of volts, the extra charge is like a teacup poured into an ocean. Fractions of micro-coulombs. The difference is far too small to be seen.

To get some visual/intuitive insight on this, go play with Red and Green Electricity, colored plastic sheets representing the populations of mobile charges within conductors. Red laid on green gives black, no color. Yet the green can still move relative to the red, even though the green has been cancelled out. In general, electric currents are flows of this "uncharged charge."


ATOMS HAVE EQUAL NUMBERS OF ELECTRONS AND PROTONS? Not in conductors!

Many students misunderstand how electric circuits work. One reason for this is that they think the electrons in a metal are trapped on individual metal atoms. They also think that an applied voltage is needed to "free" the electrons and to change metal into a conductor. They aren't aware that the "sea" of free electrons always exists inside metal all the time. I suspect that this is part of a more general misconception that all atoms in a material are always neutral. This is wrong because all conductors contain charged, movable particles. The very definition of "conductor" is "a material which contains mobile charges." If all atoms were truly neutral, then conductors could not exist.

For example, a metal is made of positively charged atoms immersed in a sea of loose electrons. Apply a voltage to a metal, and its electrons begin flowing. Salt water is full of positive and negative ions. In our world a neutral sodium or chlorine atom is a very rare thing. Sodium atoms lack their outer electron and as a result have one too many protons, while chlorine usually has one extra electron: it's a negative ion. Glowing gases (fluorescent lights, neon signs, sparks) are full of movable electrons and movable positive ions. Metals, electrolytes, and plasmas; these three are the most common electrical conductors, and they owe their conductivity to the presence of non-neutral atoms, to movable charged particles which occur naturally.





A "CONDUCTOR" IS A MATERIAL WHICH ALLOWS CHARGE TO PASS THROUGH IT? Not exactly.

The scientist's definition of the word "conductor" is different than the one above, and the one above has problems. For example, a vacuum offers no barrier to flows of electric charges. If conductors allow charges to pass, then a vacuum should be a perfect conductor. Yet, vacuum is an insulator. Vacuum is nothing, so how can it act as a barrier to electric current? Also, there's a similar problem with electric currents in air: electric charges placed into the air can easily move along, yet air is an insulator. Or look at salt water versus oil. Oil is an insulator, while salt water is a conductor, yet neither liquid is able to halt the flow of any charges which are placed into it. How can we straighten out this paradox? Easy: remove the misconception. Instead, start using the proper definition of the word "conductor."

Incorrect definition:
Conductor - a material which allows charges to pass through itself

Better:
Conductor - a material which can support an electric current

Best:
Conductor - a material which contains movable electric charges

Here's an analogy:
Conductor - like a pipe which is already full of water
Insulator - like an empty pipe: tilt it, and nothing flows downhill
Insulator - like a pipe with frozen liquid; a pipe plugged by ice

If we place a Potential Difference across either air or a vacuum, no electric current appears. Air and vacuum are insulators. This is sensible, since there are zero (or very few )movable charges in air or vacuum, so any voltage placed across them is unable to cause an electric current. Or instead, if we place a voltage across a piece of metal or across a puddle of salt water, the charges present in those materials will flow, and an electric current will appear, since these substances are always full of movable charges, and therefore the "voltage pressure" causes their charges to flow. In metal, the outer electrons of the atoms are not bound upon individual atoms but instead can move through the material, and a voltage can drive these "liquid" electrons along. But if we stick our wires into oil, there will be no electric current, since oil does not contain movable charges.

If we were to inject charges into a vacuum, then we would have electric current in a vacuum. This is how TV picture tubes and vacuum tubes work; electrons are forcibly injected into the empty space by a hot filament. However, think about it for a second: it's no longer a vacuum when it contains a cloud of electrons! <grin> Maybe we should change their name to "electron-cloud tubes" rather than "vacuum tubes", since the electron cloud is required before there can be any conductivity in the vacuum between the plates. (But vacuum tubes already have another name, so this would just confuse things. They are called "hollow-state devices." As opposed to "solid state devices?" Nyuk nyuk.)




HUMID AIR IS CONDUCTIVE? Mistake.

Electrostatic experiments don't work very well under humid conditions. Some books state that the water vapor in the air makes the air conductive. Wrong. In reality the problem is caused by the liquid water that becomes adsorbed on surfaces of objects.

In order to make the air conductive, we'd have to fill it with movable charged particles. Evaporated water is not made of charged particles (ions,) instead it's made of neutral molecules, so the high humidity does not significantly affect the conductivity of the air. Even suspended water droplets (fog) does not significantly affect conductivity. For fog to be conductive, the individual droplets would have to possess an electric charge.

However, during humid conditions most insulators develop a surface layer of conductive water mixed with contaminants (including dissolved salts which makes this layer of water conductive.) If you find that you can't separate any charges by rubbing a balloon on your head, it's because the humid air has made the balloon and the hair very slightly damp. The air remains nonconductive, but surfaces of insulators become conductive when damp. Conductive surfaces don't separate any opposite charges when rubbed together. Cure this by warming them (drying them) with a blow-dryer. If a pair of insulators is sufficiently dry, it will "generate charge" even under very humid conditions. If conductive air were the culprit, this cure couldn't work.




8. LIGHTNING RODS DISCHARGE THE CLOUDS? Mistake.

UNDER CONSTRUCTION
Make a model "landscape", install some lightning rods on the tiny houses, then bring a "storm cloud" nearby: bring the metal sphere of a VandeGraaff Electrostatic generator over your small town. The strong electric charge on the sphere will vanish. Doesn't this prove that lightning rods can discharge a thunderstorm? Nope.

The above demonstration was thought at one time to be accurate, and this old mistake still appears in many books. In reality, lightning rods cannot remove the charge-imbalance from a thunderstorm. The scale of the typical demo is wrong. The stormcloud is a few miles up, and a few miles across, yet lightning rods on houses are microscopic: only a few feet tall. Therefore in our model the metal-sphere "cloud" should be fairly large, and "Rods" should be far less than 1mm tall and attached to a wide metal ground plate. They aren't like wires. They're more like felt-fuzz.

The typical demonstration doesn't illustrate a lightning rod, it illustrates a kilometers-tall radio tower or extremely tall office building.

Think about it: how can a tiny needle affect cubic kilometers of strong e-field? How could the relatively tiny current from a metal rod discharge a cloud that's over 1KM away. It can't! To do so, it would have to emit a hurricane wind made of ionized air. Unfortunately the lightning rod on your roof only emits about the same current as the needle in the model town: it emits a few microamperes. In other words, the scale model is not correct because the current coming from a *real* lightning rod is way too low. In order to be at the proper scale, the current in the model rod would have to be hundreds of thousands of times smaller; too small to have any effect upon the VDG machine's charge.


"ELECTRICITY IS WEIGHTLESS? Nope.

If by 'electricity' we mean the electrons, then 'electricity' is not weightless. Take a copper wire for example. Each atom weights about 115,000 times larger than the weight of an electron. If each atom supplies one electron to the "electric fluid" sea, then that sea is very light, but it is not weightless. The flowing "electricity" weighs about a hundred thousand times less than the copper metal. It's like a low pressure gas rather than like a liquid (but never forget that a gas is still matter!) One KG of copper would contain about ten milligrams of the movable electron-stuff which can flow as an electric current.




THE "TWO FLUIDS" THEORY WAS DISPROVED? Not exactly.

In the early days of electrical science, researchers were sure that there were two kinds of electricity: "vitreous electricity" and "resinous electricity," which were later named positive and negative electricity. Scientists imagined that these represented two kinds of "electric fluid" which were somehow created by rubbing various materials together. Ben Franklin proposed a different concept: he imagined that there was only one kind of electric fluid, positive electricity, and believed that "negative electricity" was simply a lack of electric fluid.

Some scientists objected to Franklin's idea. They rightly pointed out that, if Franklin were correct, then matter itself must be made up of negative electricity, otherwise a rubber rod wouldn't become negative when Franklin's electric fluid was removed from it. They noticed that Franklin was not proposing a single kind of electric stuff. Instead Franklin was saying that two opposite kinds of electricity exist, but only one of them is a movable "fluid." The other kind would be solidly connected to the material of an object.

In hindsight we can see that Franklin was wrong. During electric currents in batteries, in salt water, or in human flesh, the electric current is a flow of both positive and negative ions moving in opposite directions. Two flows of "electricity" take place in the same conductor. In your brain and nervous system, electric current is a flow of positive and negative atoms going in opposite directions. During electric currents in neon signs, in sparks, lightning, etc., there is a flow of both positive ions and electrons. The same is true for liquid metals. And when two materials are rubbed together, sometimes positive or negative ions are transferred, and sometimes electrons are transferred. In Franklin's language, two electric fluids do indeed exist, and Franklin's "one fluid" theory is wrong.

Franklin was somewhat correct about two things. He was right about electric current in solid (non-liquid) metals. During electric currents in wires, it's the negative "electric fluid" which flows along, while the positive stuff behaves as an "electric solid" and cannot flow. But melt the metal and this frees up the positive atoms so that they can flow too. Also, Franklin was right in suspecting that, in some situations, "positive electricity" and "negative electricity" differ greatly in mass. Protons are about 1800 times heavier than electrons, and positive ions heavier still. But when electric current is a flow of ions alone, the negative and positive ions can be very similar in mass, or the negative ions can even be far heavier than the positive.

The complexity of electric charge was far greater than Franklin and his contemporaries knew. Franklin was right about metals, but he was wrong about conductivity in general. Modern science recognizes that positive particles can flow, and recognizes the existence of both positrons and electrons, therefore it rejects Franklin's "one fluid" theory of electricity.


ELECTRIC ENERGY TRAVELS INSIDE OF WIRES? Wrong.

When an electric company's distant generator lights up your lamps, the electric energy travels along the power lines at almost the speed of light. Most K-12 textbooks teach that the energy is trapped inside of electrons, and these electrons flow inside the wires. Doesn't this mean that electrical energy flows INSIDE the metal wires? Nope, since electrical energy is not trapped inside electrons. Instead the energy is made of invisible magnetic fields and electric fields which surround the electrons, and these fields surround the wires. Electrons don't flow fast like the energy does, instead they ooze along slowly to produce an electric current. But how can electrons flow slowly if the energy flows fast? It's because the energy can leap from electron to electron. Indeed, the energy is connected to a whole vast population of electrons in the wire, and it isn't attached to any single one.

Is this confusing? Here's another way to see it. During an electric current, the wires become surrounded with magnetic field. This field IS the electrical energy. Also, whenever a pair of wires is connected to a battery or generator, the two wires become oppositely charged, and they become surrounded with an invisible electrostatic field. This field IS the electrical energy. Magnetic and electric fields exist in the empty space surrounding your lamp cord, and these fields contain the "wattage", they contain the flow of electrical energy that powers the light bulb. Electric and magnetic fields together are "Electromagnetism," the same kind of energy as radio waves and light. Those "EMFs" that people worry about; the invisible "EMFs" that surround wires and exist invisibly in our homes... that's the electromagnetic energy which lights our lights and runs our appliances. It certainly makes sense that it travels at the same speed as radio waves and light, since it's made of the exact same "stuff."

So, what would electrical energy look like if we could see it? Here are some simple drawings I made. Whenever you plug in a light bulb, the energy that flows along the lamp cord is like a fuzzy sausage a couple of inches thick. It follows the two wires of the cord, then it dives into the thin filament of the light bulb. Is this a very strange description of "electricity?" Yes and no. It's the same description taught to advanced engineers in their courses on waveguides and radio physics. It's also taught to university physics students, especially if they read chapter 27 of The Feynman Lectures. But it's not so terribly advanced, instead it's just unfamiliar, because so few people know about it or discuss it.


STORM CLOUDS ARE ELECTRIFIED BY FRICTION? No.

Some books claim that the separated charges in thunderstorms come about because the clouds rub against each other, or because the falling rain rubs against the air. This is not correct. In fact, the true explanation for storm electrification is unknown. There are several possible explanations, but none of them has yet been accepted by scientists, and all the theories have problems. Here's one current theory:
In a mixture of rain and half-melted hail, the ice and water become oppositely electrified through contact. The large hail then falls faster than the small raindrops and spray. Two large regions appear in the cloud, a lower one that's made of hail, and an upper one that's made of rain. These regions contain opposite imbalances of electric charge.
So, what caused the clouds to become electrified? Contact between dissimilar materials, followed by wide separation of those materials.

BEN FRANKLIN'S KITE WAS STRUCK BY LIGHTNING? Never happened!

Many people believe that Ben Franklin's kite was hit by a lightning bolt, and think that this was how he proved that lightning was electrical. A number of books and even some encyclopedias say the same thing. They are wrong. When lightning strikes a kite, the spreading electric currents in the ground can kill anyone standing nearby, to say nothing of the person holding the string! So what did Franklin actually do? He showed that a kite would collect a tiny bit of electric charge out of the sky during a thunderstorm. Electric leakage through the air caused his kite and string to become electrified and so the hairs on the twine stood outwards. Twine is slightly conductive, so the imbalanced charge spread to all parts of the kite string. Franklin used the twine to electrify a metal key, and tiny sparks could then be drawn from the key. (He used a metal object because sparks cannot be directly drawn from the twine, it's not conductive enough.) This suggested that some stormclouds carry strong electrical net-charge. It implied that lightning was just a large electric spark.

The common belief that Franklin easily survived a lightning strike is not just wrong, it is dangerous: it may convince kids that it's OK to duplicate the kite experiment as long as they "protect" themselves by holding a silk ribbon. Make no mistake, Franklin's experiment was extremely dangerous, and if lightning had actually hit his kite, he certainly would have been killed.


ELECTRICITY IS MADE OF ELECTRONS? Wrong.

Other things flow besides electrons. It's true that electric current in metals is a flow of electrons. But there are many other conductors besides metals, and in many of them the currents are not caused by moving electrons. Electric currents can also appear in electrolytes or in plasma. During electric shocks, no electrons flowed through your skin or body!

When an electric current creates the glowing plasma within a neon sign or electric spark, electrons flow in one direction, while positively charged atoms flow in the other. Yes, there is an electron flow in the glowing gas, but part of the total electric current is also made of moving positive atoms which flow the other way. (And if the discharge was in hydrogen gas, the positive current is a proton-flow.)

Also there can be electric currents where no electrons flow at all. The currents in electrolytes (examples: batteries, dirt, and human flesh) are entirely composed of flows of ions: electrified atoms. In acids, half the flowing ions are +H ions, also called "protons." Bare electrons can't exist in battery electrolytes or within salt water. In electrolytes, half of the charged atoms have more protons than electrons, and the other half has more electrons than protons. During a current, the opposite-charged atoms move in opposite directions. For example, when an electric current is passing through the inside of a a battery, it is not made of moving electrons, it is made of moving electrolyte atoms (ions), and each atom carries a charge imbalance. The positive atoms flow one way, and the negative atoms flow the other. Specifically in battery acid, the major portion of the current is a flow of protons. (Students often miss this basic fact, since in chemistry classrooms, moving protons are called "+H positive hydrogen ions.)

The same effect happens when an electric current passes through the damp earth, or through the ocean, or through your body. All these are non-electron currents. If you receive an electric shock, no electrons flowed inside you. These electric currents are flows of atoms. All the electric currents in your brain and nerves are composed of moving chloride, sodium and potassium atoms. No electrons. (LOTS MORE)

See also:


 
 






http://amasci.com/miscon/eleca.html
Electrica Technology Inc. and Linkedin
Created and maintained byBill Beaty. Mail me at: .
View My Stats